Textbook Proposition on Product of Real Analytic Functions












3












$begingroup$



Let
begin{align*}
sumlimits_{j=0}^infty a_j (x-c)^j && sumlimits_{j=0}^infty b_j (x-c)^j \
end{align*}



be two power series with intervals of convergence $mathcal{C}_1$ and $mathcal{C}_2$ centered on at $c$. Let $f_1(x)$ be the function defined by the first series on $mathcal{C}_1$ and $f_2(x)$ the function defined by the second series on $mathcal{C}_2$. Then, on their common domain $mathcal{C} = mathcal{C}_1 cap mathcal{C}_2$, it holds that
begin{align*}
f(x) cdot g(x) &= sumlimits_{m=0}^infty sumlimits_{j+k=m} (a_j cdot b_k) (x-c)^m \
end{align*}



Proof: Let
begin{align*}
A_N &= sumlimits_{j=0}^N a_j (x-c)^j & B_N &= sumlimits_{j=0}^N b_j (x-c)^j \
end{align*}



be respectively, the Nth partial sums of the power series that define $f$ and $g$. Let
begin{align*}
D_N &= sumlimits_{m=0}^N sumlimits_{j+k=m} (a_j cdot b_k) (x-c)^m &
R_N &= sumlimits_{j=N+1}^infty b_j (x-c)^j \
end{align*}



We have:
begin{align*}
D_N &= a_0 B_N + a_1 (x-c) B_{N-1} + cdots + a_N (x-c)^N B_0 \
&= a_0 (g(x) - R_N) + a_1 (x-c) (g(x) - R_{N-1}) + cdots + a_N (x-c)^N (g(x) - R_0) \
end{align*}

[snip]




I've transcribed the above from my Real Analysis textbook and stopped at the part I'm having trouble understanding. How do you get from the first definition of $D_N$ to the second one? They seem like different equations. I've tried to do sequence manipulations to get from one definition to the other, but I've not had success.










share|cite|improve this question









$endgroup$

















    3












    $begingroup$



    Let
    begin{align*}
    sumlimits_{j=0}^infty a_j (x-c)^j && sumlimits_{j=0}^infty b_j (x-c)^j \
    end{align*}



    be two power series with intervals of convergence $mathcal{C}_1$ and $mathcal{C}_2$ centered on at $c$. Let $f_1(x)$ be the function defined by the first series on $mathcal{C}_1$ and $f_2(x)$ the function defined by the second series on $mathcal{C}_2$. Then, on their common domain $mathcal{C} = mathcal{C}_1 cap mathcal{C}_2$, it holds that
    begin{align*}
    f(x) cdot g(x) &= sumlimits_{m=0}^infty sumlimits_{j+k=m} (a_j cdot b_k) (x-c)^m \
    end{align*}



    Proof: Let
    begin{align*}
    A_N &= sumlimits_{j=0}^N a_j (x-c)^j & B_N &= sumlimits_{j=0}^N b_j (x-c)^j \
    end{align*}



    be respectively, the Nth partial sums of the power series that define $f$ and $g$. Let
    begin{align*}
    D_N &= sumlimits_{m=0}^N sumlimits_{j+k=m} (a_j cdot b_k) (x-c)^m &
    R_N &= sumlimits_{j=N+1}^infty b_j (x-c)^j \
    end{align*}



    We have:
    begin{align*}
    D_N &= a_0 B_N + a_1 (x-c) B_{N-1} + cdots + a_N (x-c)^N B_0 \
    &= a_0 (g(x) - R_N) + a_1 (x-c) (g(x) - R_{N-1}) + cdots + a_N (x-c)^N (g(x) - R_0) \
    end{align*}

    [snip]




    I've transcribed the above from my Real Analysis textbook and stopped at the part I'm having trouble understanding. How do you get from the first definition of $D_N$ to the second one? They seem like different equations. I've tried to do sequence manipulations to get from one definition to the other, but I've not had success.










    share|cite|improve this question









    $endgroup$















      3












      3








      3


      0



      $begingroup$



      Let
      begin{align*}
      sumlimits_{j=0}^infty a_j (x-c)^j && sumlimits_{j=0}^infty b_j (x-c)^j \
      end{align*}



      be two power series with intervals of convergence $mathcal{C}_1$ and $mathcal{C}_2$ centered on at $c$. Let $f_1(x)$ be the function defined by the first series on $mathcal{C}_1$ and $f_2(x)$ the function defined by the second series on $mathcal{C}_2$. Then, on their common domain $mathcal{C} = mathcal{C}_1 cap mathcal{C}_2$, it holds that
      begin{align*}
      f(x) cdot g(x) &= sumlimits_{m=0}^infty sumlimits_{j+k=m} (a_j cdot b_k) (x-c)^m \
      end{align*}



      Proof: Let
      begin{align*}
      A_N &= sumlimits_{j=0}^N a_j (x-c)^j & B_N &= sumlimits_{j=0}^N b_j (x-c)^j \
      end{align*}



      be respectively, the Nth partial sums of the power series that define $f$ and $g$. Let
      begin{align*}
      D_N &= sumlimits_{m=0}^N sumlimits_{j+k=m} (a_j cdot b_k) (x-c)^m &
      R_N &= sumlimits_{j=N+1}^infty b_j (x-c)^j \
      end{align*}



      We have:
      begin{align*}
      D_N &= a_0 B_N + a_1 (x-c) B_{N-1} + cdots + a_N (x-c)^N B_0 \
      &= a_0 (g(x) - R_N) + a_1 (x-c) (g(x) - R_{N-1}) + cdots + a_N (x-c)^N (g(x) - R_0) \
      end{align*}

      [snip]




      I've transcribed the above from my Real Analysis textbook and stopped at the part I'm having trouble understanding. How do you get from the first definition of $D_N$ to the second one? They seem like different equations. I've tried to do sequence manipulations to get from one definition to the other, but I've not had success.










      share|cite|improve this question









      $endgroup$





      Let
      begin{align*}
      sumlimits_{j=0}^infty a_j (x-c)^j && sumlimits_{j=0}^infty b_j (x-c)^j \
      end{align*}



      be two power series with intervals of convergence $mathcal{C}_1$ and $mathcal{C}_2$ centered on at $c$. Let $f_1(x)$ be the function defined by the first series on $mathcal{C}_1$ and $f_2(x)$ the function defined by the second series on $mathcal{C}_2$. Then, on their common domain $mathcal{C} = mathcal{C}_1 cap mathcal{C}_2$, it holds that
      begin{align*}
      f(x) cdot g(x) &= sumlimits_{m=0}^infty sumlimits_{j+k=m} (a_j cdot b_k) (x-c)^m \
      end{align*}



      Proof: Let
      begin{align*}
      A_N &= sumlimits_{j=0}^N a_j (x-c)^j & B_N &= sumlimits_{j=0}^N b_j (x-c)^j \
      end{align*}



      be respectively, the Nth partial sums of the power series that define $f$ and $g$. Let
      begin{align*}
      D_N &= sumlimits_{m=0}^N sumlimits_{j+k=m} (a_j cdot b_k) (x-c)^m &
      R_N &= sumlimits_{j=N+1}^infty b_j (x-c)^j \
      end{align*}



      We have:
      begin{align*}
      D_N &= a_0 B_N + a_1 (x-c) B_{N-1} + cdots + a_N (x-c)^N B_0 \
      &= a_0 (g(x) - R_N) + a_1 (x-c) (g(x) - R_{N-1}) + cdots + a_N (x-c)^N (g(x) - R_0) \
      end{align*}

      [snip]




      I've transcribed the above from my Real Analysis textbook and stopped at the part I'm having trouble understanding. How do you get from the first definition of $D_N$ to the second one? They seem like different equations. I've tried to do sequence manipulations to get from one definition to the other, but I've not had success.







      real-analysis power-series analytic-functions






      share|cite|improve this question













      share|cite|improve this question











      share|cite|improve this question




      share|cite|improve this question










      asked Dec 5 '18 at 15:38









      clayclay

      774415




      774415






















          2 Answers
          2






          active

          oldest

          votes


















          1












          $begingroup$

          We start with the first representation of $D_n$ and obtain the second one.




          We obtain
          begin{align*}
          color{blue}{D_n}&color{blue}{=sum_{m=0}^Nleft(sum_{{j+k=m}atop{j,kgeq 0}} a_jb_kright)(x-c)^m}\
          &=sum_{m=0}^Nleft(sum_{j=0}^m a_jb_{m-j}right)(x-c)^mtag{1}\
          &=sum_{0leq jleq mleq N} a_jb_{m-j}(x-c)^mtag{2}\
          &=sum_{j=0}^Nsum_{m=j}^N a_jb_{m-j}(x-c)^mtag{3}\
          &=sum_{j=0}^Na_jsum_{m=0}^{N-j}b_m(x-c)^{m+j}tag{4}\
          &=sum_{j=0}^N a_j(x-c)^jsum_{m=0}^{N-j}b_m(x-c)^mtag{5}\
          &,,color{blue}{=sum_{j=0}^N a_j(x-c)^jB_{N-j}}tag{6}
          end{align*}

          and we finally got the second representation of $D_n$.




          Comment:




          • In (1) we eliminate $k$ by substituting $kto m-j$.


          • In (2) we write the index region somewhat more conveniently.


          • In (3) we exchange the order of summation.


          • In (4) we shift the index $m$ to start with $m=0$.


          • In (5) we factor out the terms which are not dependent on $m$.




          We also obtain for $0leq jleq N$
          begin{align*}
          color{blue}{g(x)}&=sum_{k=0}^infty b_k(x-c)^k\
          &=sum_{k=0}^{N-j} b_k(x-c)^k+sum_{k=N-j+1}^infty b_k(x-c)^k\
          &,,color{blue}{=B_{N-j}+R_{N-j}}
          end{align*}

          and the last line of OP's post follows from (6) by replacing $B_{N-j}=g(x)-R_{N-j}$.







          share|cite|improve this answer











          $endgroup$













          • $begingroup$
            Amazing. Except that I believe that (3) should start with $sum_{j=0}^N$ rather than $sum_{m=0}^N$
            $endgroup$
            – clay
            Dec 5 '18 at 22:02










          • $begingroup$
            @clay: You're welcome. Typo corrected. Thanks.
            $endgroup$
            – Markus Scheuer
            Dec 5 '18 at 22:11



















          1












          $begingroup$

          You can actually prove this by induction




          Works for $N = 1$




          begin{eqnarray}
          D_1 &=& a_0b_0 + (a_0b_1 + a_1 b_0)(x - c) \ &=& a_0 [b_0 + b_1(x - c)] + a_1[b_0(x - c)] \
          &=& a_0 B_1 + a_1 B_0
          end{eqnarray}




          Assume that it works for $N - 1$




          $$
          sum_{m = 0}^{N - 1}sum_{j + k = m}a_j b_k(x - c)^m = a_0 B_{N - 1}
          + a_1(x - c) B_{N - 2} + cdots + a_{N-1}(x - c)^{N-1}B_0 tag{1}
          $$




          Now let's prove it for $N$




          begin{eqnarray}
          sum_{m = 0}^{N}sum_{j + k = m}a_j b_k(x - c)^m &=& sum_{m = 0}^{N - 1} sum_{j + k = m}a_j b_k(x - c)^m + sum_{j + k = N}a_j b_k(x - c)^N \
          &stackrel{(1)}{=}& a_0 B_{N - 1}
          + a_1(x - c) B_{N - 2} + cdots + a_{N-1}(x - c)^{N-1}B_0 \
          && + [a_0 b_N + a_1 b_{N - 1} + cdots + a_N b_0](x - c)^N \
          &=& a_0 left[ B_{N - 1} + b_N(x - c)^N right] + a_1(x - c) left[ B_{N - 2} + b_{N - 1}(x - c)^{N - 1} right] \
          && + cdots + a_{N}(x - c)^{N}[b_0] \
          &=&a_0 B_N + a_1(x - c) B_{N - 1} + cdots + a_{N}(x - c)^{N}B_0 tag{2}
          end{eqnarray}






          share|cite|improve this answer









          $endgroup$













          • $begingroup$
            Awesome! Thank you so much!
            $endgroup$
            – clay
            Dec 5 '18 at 22:03











          Your Answer





          StackExchange.ifUsing("editor", function () {
          return StackExchange.using("mathjaxEditing", function () {
          StackExchange.MarkdownEditor.creationCallbacks.add(function (editor, postfix) {
          StackExchange.mathjaxEditing.prepareWmdForMathJax(editor, postfix, [["$", "$"], ["\\(","\\)"]]);
          });
          });
          }, "mathjax-editing");

          StackExchange.ready(function() {
          var channelOptions = {
          tags: "".split(" "),
          id: "69"
          };
          initTagRenderer("".split(" "), "".split(" "), channelOptions);

          StackExchange.using("externalEditor", function() {
          // Have to fire editor after snippets, if snippets enabled
          if (StackExchange.settings.snippets.snippetsEnabled) {
          StackExchange.using("snippets", function() {
          createEditor();
          });
          }
          else {
          createEditor();
          }
          });

          function createEditor() {
          StackExchange.prepareEditor({
          heartbeatType: 'answer',
          autoActivateHeartbeat: false,
          convertImagesToLinks: true,
          noModals: true,
          showLowRepImageUploadWarning: true,
          reputationToPostImages: 10,
          bindNavPrevention: true,
          postfix: "",
          imageUploader: {
          brandingHtml: "Powered by u003ca class="icon-imgur-white" href="https://imgur.com/"u003eu003c/au003e",
          contentPolicyHtml: "User contributions licensed under u003ca href="https://creativecommons.org/licenses/by-sa/3.0/"u003ecc by-sa 3.0 with attribution requiredu003c/au003e u003ca href="https://stackoverflow.com/legal/content-policy"u003e(content policy)u003c/au003e",
          allowUrls: true
          },
          noCode: true, onDemand: true,
          discardSelector: ".discard-answer"
          ,immediatelyShowMarkdownHelp:true
          });


          }
          });














          draft saved

          draft discarded


















          StackExchange.ready(
          function () {
          StackExchange.openid.initPostLogin('.new-post-login', 'https%3a%2f%2fmath.stackexchange.com%2fquestions%2f3027214%2ftextbook-proposition-on-product-of-real-analytic-functions%23new-answer', 'question_page');
          }
          );

          Post as a guest















          Required, but never shown

























          2 Answers
          2






          active

          oldest

          votes








          2 Answers
          2






          active

          oldest

          votes









          active

          oldest

          votes






          active

          oldest

          votes









          1












          $begingroup$

          We start with the first representation of $D_n$ and obtain the second one.




          We obtain
          begin{align*}
          color{blue}{D_n}&color{blue}{=sum_{m=0}^Nleft(sum_{{j+k=m}atop{j,kgeq 0}} a_jb_kright)(x-c)^m}\
          &=sum_{m=0}^Nleft(sum_{j=0}^m a_jb_{m-j}right)(x-c)^mtag{1}\
          &=sum_{0leq jleq mleq N} a_jb_{m-j}(x-c)^mtag{2}\
          &=sum_{j=0}^Nsum_{m=j}^N a_jb_{m-j}(x-c)^mtag{3}\
          &=sum_{j=0}^Na_jsum_{m=0}^{N-j}b_m(x-c)^{m+j}tag{4}\
          &=sum_{j=0}^N a_j(x-c)^jsum_{m=0}^{N-j}b_m(x-c)^mtag{5}\
          &,,color{blue}{=sum_{j=0}^N a_j(x-c)^jB_{N-j}}tag{6}
          end{align*}

          and we finally got the second representation of $D_n$.




          Comment:




          • In (1) we eliminate $k$ by substituting $kto m-j$.


          • In (2) we write the index region somewhat more conveniently.


          • In (3) we exchange the order of summation.


          • In (4) we shift the index $m$ to start with $m=0$.


          • In (5) we factor out the terms which are not dependent on $m$.




          We also obtain for $0leq jleq N$
          begin{align*}
          color{blue}{g(x)}&=sum_{k=0}^infty b_k(x-c)^k\
          &=sum_{k=0}^{N-j} b_k(x-c)^k+sum_{k=N-j+1}^infty b_k(x-c)^k\
          &,,color{blue}{=B_{N-j}+R_{N-j}}
          end{align*}

          and the last line of OP's post follows from (6) by replacing $B_{N-j}=g(x)-R_{N-j}$.







          share|cite|improve this answer











          $endgroup$













          • $begingroup$
            Amazing. Except that I believe that (3) should start with $sum_{j=0}^N$ rather than $sum_{m=0}^N$
            $endgroup$
            – clay
            Dec 5 '18 at 22:02










          • $begingroup$
            @clay: You're welcome. Typo corrected. Thanks.
            $endgroup$
            – Markus Scheuer
            Dec 5 '18 at 22:11
















          1












          $begingroup$

          We start with the first representation of $D_n$ and obtain the second one.




          We obtain
          begin{align*}
          color{blue}{D_n}&color{blue}{=sum_{m=0}^Nleft(sum_{{j+k=m}atop{j,kgeq 0}} a_jb_kright)(x-c)^m}\
          &=sum_{m=0}^Nleft(sum_{j=0}^m a_jb_{m-j}right)(x-c)^mtag{1}\
          &=sum_{0leq jleq mleq N} a_jb_{m-j}(x-c)^mtag{2}\
          &=sum_{j=0}^Nsum_{m=j}^N a_jb_{m-j}(x-c)^mtag{3}\
          &=sum_{j=0}^Na_jsum_{m=0}^{N-j}b_m(x-c)^{m+j}tag{4}\
          &=sum_{j=0}^N a_j(x-c)^jsum_{m=0}^{N-j}b_m(x-c)^mtag{5}\
          &,,color{blue}{=sum_{j=0}^N a_j(x-c)^jB_{N-j}}tag{6}
          end{align*}

          and we finally got the second representation of $D_n$.




          Comment:




          • In (1) we eliminate $k$ by substituting $kto m-j$.


          • In (2) we write the index region somewhat more conveniently.


          • In (3) we exchange the order of summation.


          • In (4) we shift the index $m$ to start with $m=0$.


          • In (5) we factor out the terms which are not dependent on $m$.




          We also obtain for $0leq jleq N$
          begin{align*}
          color{blue}{g(x)}&=sum_{k=0}^infty b_k(x-c)^k\
          &=sum_{k=0}^{N-j} b_k(x-c)^k+sum_{k=N-j+1}^infty b_k(x-c)^k\
          &,,color{blue}{=B_{N-j}+R_{N-j}}
          end{align*}

          and the last line of OP's post follows from (6) by replacing $B_{N-j}=g(x)-R_{N-j}$.







          share|cite|improve this answer











          $endgroup$













          • $begingroup$
            Amazing. Except that I believe that (3) should start with $sum_{j=0}^N$ rather than $sum_{m=0}^N$
            $endgroup$
            – clay
            Dec 5 '18 at 22:02










          • $begingroup$
            @clay: You're welcome. Typo corrected. Thanks.
            $endgroup$
            – Markus Scheuer
            Dec 5 '18 at 22:11














          1












          1








          1





          $begingroup$

          We start with the first representation of $D_n$ and obtain the second one.




          We obtain
          begin{align*}
          color{blue}{D_n}&color{blue}{=sum_{m=0}^Nleft(sum_{{j+k=m}atop{j,kgeq 0}} a_jb_kright)(x-c)^m}\
          &=sum_{m=0}^Nleft(sum_{j=0}^m a_jb_{m-j}right)(x-c)^mtag{1}\
          &=sum_{0leq jleq mleq N} a_jb_{m-j}(x-c)^mtag{2}\
          &=sum_{j=0}^Nsum_{m=j}^N a_jb_{m-j}(x-c)^mtag{3}\
          &=sum_{j=0}^Na_jsum_{m=0}^{N-j}b_m(x-c)^{m+j}tag{4}\
          &=sum_{j=0}^N a_j(x-c)^jsum_{m=0}^{N-j}b_m(x-c)^mtag{5}\
          &,,color{blue}{=sum_{j=0}^N a_j(x-c)^jB_{N-j}}tag{6}
          end{align*}

          and we finally got the second representation of $D_n$.




          Comment:




          • In (1) we eliminate $k$ by substituting $kto m-j$.


          • In (2) we write the index region somewhat more conveniently.


          • In (3) we exchange the order of summation.


          • In (4) we shift the index $m$ to start with $m=0$.


          • In (5) we factor out the terms which are not dependent on $m$.




          We also obtain for $0leq jleq N$
          begin{align*}
          color{blue}{g(x)}&=sum_{k=0}^infty b_k(x-c)^k\
          &=sum_{k=0}^{N-j} b_k(x-c)^k+sum_{k=N-j+1}^infty b_k(x-c)^k\
          &,,color{blue}{=B_{N-j}+R_{N-j}}
          end{align*}

          and the last line of OP's post follows from (6) by replacing $B_{N-j}=g(x)-R_{N-j}$.







          share|cite|improve this answer











          $endgroup$



          We start with the first representation of $D_n$ and obtain the second one.




          We obtain
          begin{align*}
          color{blue}{D_n}&color{blue}{=sum_{m=0}^Nleft(sum_{{j+k=m}atop{j,kgeq 0}} a_jb_kright)(x-c)^m}\
          &=sum_{m=0}^Nleft(sum_{j=0}^m a_jb_{m-j}right)(x-c)^mtag{1}\
          &=sum_{0leq jleq mleq N} a_jb_{m-j}(x-c)^mtag{2}\
          &=sum_{j=0}^Nsum_{m=j}^N a_jb_{m-j}(x-c)^mtag{3}\
          &=sum_{j=0}^Na_jsum_{m=0}^{N-j}b_m(x-c)^{m+j}tag{4}\
          &=sum_{j=0}^N a_j(x-c)^jsum_{m=0}^{N-j}b_m(x-c)^mtag{5}\
          &,,color{blue}{=sum_{j=0}^N a_j(x-c)^jB_{N-j}}tag{6}
          end{align*}

          and we finally got the second representation of $D_n$.




          Comment:




          • In (1) we eliminate $k$ by substituting $kto m-j$.


          • In (2) we write the index region somewhat more conveniently.


          • In (3) we exchange the order of summation.


          • In (4) we shift the index $m$ to start with $m=0$.


          • In (5) we factor out the terms which are not dependent on $m$.




          We also obtain for $0leq jleq N$
          begin{align*}
          color{blue}{g(x)}&=sum_{k=0}^infty b_k(x-c)^k\
          &=sum_{k=0}^{N-j} b_k(x-c)^k+sum_{k=N-j+1}^infty b_k(x-c)^k\
          &,,color{blue}{=B_{N-j}+R_{N-j}}
          end{align*}

          and the last line of OP's post follows from (6) by replacing $B_{N-j}=g(x)-R_{N-j}$.








          share|cite|improve this answer














          share|cite|improve this answer



          share|cite|improve this answer








          edited Dec 5 '18 at 22:10

























          answered Dec 5 '18 at 21:40









          Markus ScheuerMarkus Scheuer

          62.2k459149




          62.2k459149












          • $begingroup$
            Amazing. Except that I believe that (3) should start with $sum_{j=0}^N$ rather than $sum_{m=0}^N$
            $endgroup$
            – clay
            Dec 5 '18 at 22:02










          • $begingroup$
            @clay: You're welcome. Typo corrected. Thanks.
            $endgroup$
            – Markus Scheuer
            Dec 5 '18 at 22:11


















          • $begingroup$
            Amazing. Except that I believe that (3) should start with $sum_{j=0}^N$ rather than $sum_{m=0}^N$
            $endgroup$
            – clay
            Dec 5 '18 at 22:02










          • $begingroup$
            @clay: You're welcome. Typo corrected. Thanks.
            $endgroup$
            – Markus Scheuer
            Dec 5 '18 at 22:11
















          $begingroup$
          Amazing. Except that I believe that (3) should start with $sum_{j=0}^N$ rather than $sum_{m=0}^N$
          $endgroup$
          – clay
          Dec 5 '18 at 22:02




          $begingroup$
          Amazing. Except that I believe that (3) should start with $sum_{j=0}^N$ rather than $sum_{m=0}^N$
          $endgroup$
          – clay
          Dec 5 '18 at 22:02












          $begingroup$
          @clay: You're welcome. Typo corrected. Thanks.
          $endgroup$
          – Markus Scheuer
          Dec 5 '18 at 22:11




          $begingroup$
          @clay: You're welcome. Typo corrected. Thanks.
          $endgroup$
          – Markus Scheuer
          Dec 5 '18 at 22:11











          1












          $begingroup$

          You can actually prove this by induction




          Works for $N = 1$




          begin{eqnarray}
          D_1 &=& a_0b_0 + (a_0b_1 + a_1 b_0)(x - c) \ &=& a_0 [b_0 + b_1(x - c)] + a_1[b_0(x - c)] \
          &=& a_0 B_1 + a_1 B_0
          end{eqnarray}




          Assume that it works for $N - 1$




          $$
          sum_{m = 0}^{N - 1}sum_{j + k = m}a_j b_k(x - c)^m = a_0 B_{N - 1}
          + a_1(x - c) B_{N - 2} + cdots + a_{N-1}(x - c)^{N-1}B_0 tag{1}
          $$




          Now let's prove it for $N$




          begin{eqnarray}
          sum_{m = 0}^{N}sum_{j + k = m}a_j b_k(x - c)^m &=& sum_{m = 0}^{N - 1} sum_{j + k = m}a_j b_k(x - c)^m + sum_{j + k = N}a_j b_k(x - c)^N \
          &stackrel{(1)}{=}& a_0 B_{N - 1}
          + a_1(x - c) B_{N - 2} + cdots + a_{N-1}(x - c)^{N-1}B_0 \
          && + [a_0 b_N + a_1 b_{N - 1} + cdots + a_N b_0](x - c)^N \
          &=& a_0 left[ B_{N - 1} + b_N(x - c)^N right] + a_1(x - c) left[ B_{N - 2} + b_{N - 1}(x - c)^{N - 1} right] \
          && + cdots + a_{N}(x - c)^{N}[b_0] \
          &=&a_0 B_N + a_1(x - c) B_{N - 1} + cdots + a_{N}(x - c)^{N}B_0 tag{2}
          end{eqnarray}






          share|cite|improve this answer









          $endgroup$













          • $begingroup$
            Awesome! Thank you so much!
            $endgroup$
            – clay
            Dec 5 '18 at 22:03
















          1












          $begingroup$

          You can actually prove this by induction




          Works for $N = 1$




          begin{eqnarray}
          D_1 &=& a_0b_0 + (a_0b_1 + a_1 b_0)(x - c) \ &=& a_0 [b_0 + b_1(x - c)] + a_1[b_0(x - c)] \
          &=& a_0 B_1 + a_1 B_0
          end{eqnarray}




          Assume that it works for $N - 1$




          $$
          sum_{m = 0}^{N - 1}sum_{j + k = m}a_j b_k(x - c)^m = a_0 B_{N - 1}
          + a_1(x - c) B_{N - 2} + cdots + a_{N-1}(x - c)^{N-1}B_0 tag{1}
          $$




          Now let's prove it for $N$




          begin{eqnarray}
          sum_{m = 0}^{N}sum_{j + k = m}a_j b_k(x - c)^m &=& sum_{m = 0}^{N - 1} sum_{j + k = m}a_j b_k(x - c)^m + sum_{j + k = N}a_j b_k(x - c)^N \
          &stackrel{(1)}{=}& a_0 B_{N - 1}
          + a_1(x - c) B_{N - 2} + cdots + a_{N-1}(x - c)^{N-1}B_0 \
          && + [a_0 b_N + a_1 b_{N - 1} + cdots + a_N b_0](x - c)^N \
          &=& a_0 left[ B_{N - 1} + b_N(x - c)^N right] + a_1(x - c) left[ B_{N - 2} + b_{N - 1}(x - c)^{N - 1} right] \
          && + cdots + a_{N}(x - c)^{N}[b_0] \
          &=&a_0 B_N + a_1(x - c) B_{N - 1} + cdots + a_{N}(x - c)^{N}B_0 tag{2}
          end{eqnarray}






          share|cite|improve this answer









          $endgroup$













          • $begingroup$
            Awesome! Thank you so much!
            $endgroup$
            – clay
            Dec 5 '18 at 22:03














          1












          1








          1





          $begingroup$

          You can actually prove this by induction




          Works for $N = 1$




          begin{eqnarray}
          D_1 &=& a_0b_0 + (a_0b_1 + a_1 b_0)(x - c) \ &=& a_0 [b_0 + b_1(x - c)] + a_1[b_0(x - c)] \
          &=& a_0 B_1 + a_1 B_0
          end{eqnarray}




          Assume that it works for $N - 1$




          $$
          sum_{m = 0}^{N - 1}sum_{j + k = m}a_j b_k(x - c)^m = a_0 B_{N - 1}
          + a_1(x - c) B_{N - 2} + cdots + a_{N-1}(x - c)^{N-1}B_0 tag{1}
          $$




          Now let's prove it for $N$




          begin{eqnarray}
          sum_{m = 0}^{N}sum_{j + k = m}a_j b_k(x - c)^m &=& sum_{m = 0}^{N - 1} sum_{j + k = m}a_j b_k(x - c)^m + sum_{j + k = N}a_j b_k(x - c)^N \
          &stackrel{(1)}{=}& a_0 B_{N - 1}
          + a_1(x - c) B_{N - 2} + cdots + a_{N-1}(x - c)^{N-1}B_0 \
          && + [a_0 b_N + a_1 b_{N - 1} + cdots + a_N b_0](x - c)^N \
          &=& a_0 left[ B_{N - 1} + b_N(x - c)^N right] + a_1(x - c) left[ B_{N - 2} + b_{N - 1}(x - c)^{N - 1} right] \
          && + cdots + a_{N}(x - c)^{N}[b_0] \
          &=&a_0 B_N + a_1(x - c) B_{N - 1} + cdots + a_{N}(x - c)^{N}B_0 tag{2}
          end{eqnarray}






          share|cite|improve this answer









          $endgroup$



          You can actually prove this by induction




          Works for $N = 1$




          begin{eqnarray}
          D_1 &=& a_0b_0 + (a_0b_1 + a_1 b_0)(x - c) \ &=& a_0 [b_0 + b_1(x - c)] + a_1[b_0(x - c)] \
          &=& a_0 B_1 + a_1 B_0
          end{eqnarray}




          Assume that it works for $N - 1$




          $$
          sum_{m = 0}^{N - 1}sum_{j + k = m}a_j b_k(x - c)^m = a_0 B_{N - 1}
          + a_1(x - c) B_{N - 2} + cdots + a_{N-1}(x - c)^{N-1}B_0 tag{1}
          $$




          Now let's prove it for $N$




          begin{eqnarray}
          sum_{m = 0}^{N}sum_{j + k = m}a_j b_k(x - c)^m &=& sum_{m = 0}^{N - 1} sum_{j + k = m}a_j b_k(x - c)^m + sum_{j + k = N}a_j b_k(x - c)^N \
          &stackrel{(1)}{=}& a_0 B_{N - 1}
          + a_1(x - c) B_{N - 2} + cdots + a_{N-1}(x - c)^{N-1}B_0 \
          && + [a_0 b_N + a_1 b_{N - 1} + cdots + a_N b_0](x - c)^N \
          &=& a_0 left[ B_{N - 1} + b_N(x - c)^N right] + a_1(x - c) left[ B_{N - 2} + b_{N - 1}(x - c)^{N - 1} right] \
          && + cdots + a_{N}(x - c)^{N}[b_0] \
          &=&a_0 B_N + a_1(x - c) B_{N - 1} + cdots + a_{N}(x - c)^{N}B_0 tag{2}
          end{eqnarray}







          share|cite|improve this answer












          share|cite|improve this answer



          share|cite|improve this answer










          answered Dec 5 '18 at 17:48









          caveraccaverac

          14.6k31130




          14.6k31130












          • $begingroup$
            Awesome! Thank you so much!
            $endgroup$
            – clay
            Dec 5 '18 at 22:03


















          • $begingroup$
            Awesome! Thank you so much!
            $endgroup$
            – clay
            Dec 5 '18 at 22:03
















          $begingroup$
          Awesome! Thank you so much!
          $endgroup$
          – clay
          Dec 5 '18 at 22:03




          $begingroup$
          Awesome! Thank you so much!
          $endgroup$
          – clay
          Dec 5 '18 at 22:03


















          draft saved

          draft discarded




















































          Thanks for contributing an answer to Mathematics Stack Exchange!


          • Please be sure to answer the question. Provide details and share your research!

          But avoid



          • Asking for help, clarification, or responding to other answers.

          • Making statements based on opinion; back them up with references or personal experience.


          Use MathJax to format equations. MathJax reference.


          To learn more, see our tips on writing great answers.




          draft saved


          draft discarded














          StackExchange.ready(
          function () {
          StackExchange.openid.initPostLogin('.new-post-login', 'https%3a%2f%2fmath.stackexchange.com%2fquestions%2f3027214%2ftextbook-proposition-on-product-of-real-analytic-functions%23new-answer', 'question_page');
          }
          );

          Post as a guest















          Required, but never shown





















































          Required, but never shown














          Required, but never shown












          Required, but never shown







          Required, but never shown

































          Required, but never shown














          Required, but never shown












          Required, but never shown







          Required, but never shown







          Popular posts from this blog

          Biblatex bibliography style without URLs when DOI exists (in Overleaf with Zotero bibliography)

          ComboBox Display Member on multiple fields

          Is it possible to collect Nectar points via Trainline?